LSAT and Law School Admissions Forum

Get expert LSAT preparation and law school admissions advice from PowerScore Test Preparation.

User avatar
 Dave Killoran
PowerScore Staff
  • PowerScore Staff
  • Posts: 5852
  • Joined: Mar 25, 2011
|
#25343
Complete Question Explanation
(The complete setup for this game can be found here: lsat/viewtopic.php?t=6424)

The correct answer choice is (E)

The question stem specifies that six applicants are interviewed (F, G, and 4 others), and that three applicants are hired. Since G must be interviewed, it follows that J and L must also be interviewed (first two rules). Clearly, then, four of the six spaces in the "interviewed" group are taken up by F, G, J, and L.
O97_Game_#2_#9_diagram 1.png
Since F, G, J, and L take up four of the six interview spaces, only two of K, M, and O can be interviewed:
O97_Game_#2_#9_diagram 2.png
The information above is all that can be determined from the conditions in the question stem.

Answer choices (A), (B), and (D) can all be eliminated by applying the last rule that states when “M is hired, and L is interviewed, O is hired.” Because M appears in each of those three answer choices but O does not appear in any of those answer choices, they are all incorrect.

Answer choice (C) is incorrect because it does not allow each hired applicant to be interviewed. The answer lists K, L, and O as the hirees, and so each must be interviewed. L was already established as being interviewed, and K and O must then occupy the remaining two spaces in the interview group, and M is left out of the interview group. This presents a problem because according to the fifth rule, when K is hired then M must be interviewed. Thus, this answer creates an unworkable group and is incorrect.

Thus, answer choice (E) is proven correct by process of elimination.
You do not have the required permissions to view the files attached to this post.

Get the most out of your LSAT Prep Plus subscription.

Analyze and track your performance with our Testing and Analytics Package.